Son İletiler

Sayfa: [1] 2 3 ... 10
1
Analiz-Cebir / Ynt: Genelleştirilmiş IMO Shortlist 2001 #A.3
« Son İleti Gönderen: Hüseyin Yiğit EMEKÇİ Mart 27, 2024, 06:18:42 ös »
$$k=1$$
verildiğinde problem IMO Shortlist 2001 #A.3'e dönüşür ve $LHS<\sqrt{n}$ eşitsizliğini elde ederiz.
2
Analiz-Cebir / Genelleştirilmiş IMO Shortlist 2001 #A.3
« Son İleti Gönderen: Hüseyin Yiğit EMEKÇİ Mart 27, 2024, 06:17:32 ös »
Genelleştirme 1
Her $x_1,x_2,\cdots,x_n$ reeli ve $k\geq 1$ tam sayısı için


$$\dfrac{x_1}{1+x_1^{2k}}+\dfrac{x_2}{1+x_2^{2k}}+\cdots+\dfrac{x_n}{1+x_n^{2k}}<\sqrt{n^{2k-1}}$$


olduğunu gösteriniz.
3
Analiz-Cebir / IMO Shortlist 2001 #A.3
« Son İleti Gönderen: Hüseyin Yiğit EMEKÇİ Mart 27, 2024, 06:14:15 ös »
Her $x_1,x_2,\cdots,x_n$ reeli için


$$\dfrac{x_1}{1+x_1^2}+\dfrac{x_2}{1+x_2^2}+\cdots+\dfrac{x_n}{1+x_n^2}<\sqrt{n}$$


olduğunu gösteriniz.
4
2007 / Ynt: Tübitak Lise 1. Aşama 2007 Soru 06
« Son İleti Gönderen: Metin Can Aydemir Mart 27, 2024, 10:34:39 öö »
$221=13\cdot 17$ olduğundan $n \geq 13$ için $n!(2n+1)$ ile $221$ aralarında asal olmaz. O halde $n \leq 12$ dir. Ancak $n=8$ için $2n+1=17$ olduğundan bu halde de $n!(2n+1)$ ile $221$ aralarında asal olmaz. Sonuç olarak $12-1=11$ tane $n$ değeri bulunur.

$n=6$ için de $2n+1=13$ olacağından $n!(2n+1)$ ile $221$ aralarında asal değildir.
5
Analiz-Cebir / Ynt: Genelleştirilmiş APMO 2005 #2
« Son İleti Gönderen: Hüseyin Yiğit EMEKÇİ Mart 25, 2024, 06:33:30 ös »
$$\lambda=2$$
verildiğinde problem APMO 2005 #2'e dönüşür ve minimum değer
$$\dfrac{4}{\dfrac{\lambda^6+8}{3\lambda^2\left(3\lambda^2+2\right)}+2}=\dfrac{4}{3}$$
olarak elde edilir.
6
Analiz-Cebir / Genelleştirilmiş APMO 2005 #2
« Son İleti Gönderen: Hüseyin Yiğit EMEKÇİ Mart 25, 2024, 06:26:17 ös »
Tüm $abc=\lambda^3$ koşulunu sağlayan pozitif reel sayılar $a,b,c$ için


$$\dfrac{a^2}{\sqrt{\left(1+a^3\right)\left(1+b^3\right)}}+\dfrac{b^2}{\sqrt{\left(1+b^3\right)\left(1+c^3\right)}}+\dfrac{c^2}{\sqrt{\left(1+c^3\right)\left(1+a^3\right)}}\geq \dfrac{4}{\dfrac{\lambda^6+8}{3\lambda^2\left(3\lambda^2+2\right)}+2}$$


eşitsizliğinin çalıştığını gösteriniz.

7
Kombinatorik / Ynt: Çokgende Alt Bölge Sayısı
« Son İleti Gönderen: alpercay Mart 25, 2024, 12:37:39 ös »
Çok bilinen ve Geomaniada olması gerektiğini düşündüğüm bu sorunun bildiğim çözümü şu şekilde:

Başlangıçta $1$ bölge var. Çokgenin her bir köşegeni çizildiğinde bölge sayısının $1$ arttığını gözlemleyebiliriz.  Ayrıca iki köşegen birbirini kestiğinde de bölge sayısı yine $1$ artar. Maksimum bölge sayısını hesaplayacaksak iki köşegenin kesim noktasından başka köşegen geçmediğini varsayalım. Bu durumda bölge sayısı $$1+C(n,2)-n+C(n,4)$$ olarak bulunur.

Bir çember üzerinde sırası ile $1,2,3,..$ nokta alınıp bunların ikişer ikişer birleştirilmesiyle çemberin kaç bölgeye ayrıldığı sorulsaydı (örüntü $1,2,4,8,16,...$ diye devam ediyor ama $6$ nokta için beklenen değer gelmiyor. Örüntüye güven olmaz yani) bölge sayısı çember ve kirişler arasında oluşan bölgelerle $n$ kadar daha artacağından istenen yanıt $$1+C(n,2)+C(n,4)$$ veya açık olarak $$\dfrac{n^4-6n^3+23n^2-18n+24}{24}$$ olurdu.
8
Analiz-Cebir / Genelleştirilmiş Romanya JBMO TST 2019 #1.2
« Son İleti Gönderen: Hüseyin Yiğit EMEKÇİ Mart 25, 2024, 08:14:27 öö »
Genelleştirme 1
Her $a_1,a_2,\cdots,a_n$ reeli için


$$\dfrac{\left(\sum\limits_{cyc}{x_1}\right)^k}{\prod\limits_{cyc- j}{\left(\left(2kx_j\right)^{2k}+4k-1\right)}}\geq \dfrac{1}{\left(4k\right)^{2k}}$$


olduğunu gösteriniz.
9
Analiz-Cebir / Ynt: Romanya JBMO TST 2019 #1.2
« Son İleti Gönderen: Hüseyin Yiğit EMEKÇİ Mart 25, 2024, 12:57:28 öö »
Maksimum değeri elde etmek için $a+b>0$ olduğu koşulu incelemek yeterlidir. Cauchy'den
$$\left(4a^2+1+2\right)\left(1+4b^2+2\right)\overbrace{\geq}^{Cauchy} 4\left(a+b+1\right)^2\overbrace{\geq}^{AM-GM} 16\left(a+b\right)$$
olduğunu söyleyebiliriz. O zaman ifademizin maksimum değerini
$$\dfrac{a+b}{\left(4a^2+3\right)\left(4b^2+3\right)}\leq \dfrac{a+b}{16(a+b)}=\dfrac{1}{16}$$
olarak tayin ederiz.
10
Analiz-Cebir / Romanya JBMO TST 2019 #1.2 {çözüldü}
« Son İleti Gönderen: Hüseyin Yiğit EMEKÇİ Mart 25, 2024, 12:51:37 öö »
Her $a,b$ reeli için


$$\dfrac{a+b}{\left(4a^2+3\right)\left(4b^2+3\right)}$$


ifadesinin maksimum değerini bulunuz.
Sayfa: [1] 2 3 ... 10

SimplePortal 2.3.3 © 2008-2010, SimplePortal